Skip to main content

The nonnegative weak solution of a degenerate parabolic equation with variable exponent growth order

Abstract

A degenerate parabolic equation of the form

$$\bigl( \vert v \vert ^{\beta-1}v\bigr)_{t}= \operatorname{div} \bigl(b(x,t) \vert \nabla v \vert ^{p(x,t)-2}\nabla v \bigr)+\nabla\vec{g}\cdot\nabla\vec{\gamma}(v) $$

is considered, where \(\vec{g}=\{g^{i}(x,t)\}\), \(\vec{\gamma}(v)=\{ \gamma_{i}(v)\}\). If the diffusion coefficient \(b(x,t)\geq0\) is degenerate on the boundary, by adding some restrictions on \(b(x,t)\) and g⃗, the existence and uniqueness of weak solutions are proved. Based on the uniqueness, the stability of weak solutions can be proved without any boundary condition.

1 Introduction and the main results

Consider the degenerate parabolic problem with exponent variable growth order

$$ \bigl( \vert v \vert ^{\beta-1}v\bigr)_{t}= \operatorname{div} \bigl(b(x,t) \vert \nabla v \vert ^{p(x,t)-2}\nabla v \bigr)+ \nabla\vec{g}\cdot\nabla\vec{\gamma}(v),\quad (x,t)\in Q_{T}=\varOmega \times(0,T), $$
(1.1)

where \(b(x,t)\) and \(p(x,t)\) are \(C(\overline{Q_{T}})\) nonnegative functions, \(\vec{g}=\{g^{i}(x,t)\}\), \(\vec{\gamma}(v)=\{\gamma _{i}(v)\}\), \(\beta>0\). We denote that \(p^{-}=\min_{(x,t)\in\overline {Q_{T}}}p(x,t)>1\) and \(p^{+}=\max_{(x,t)\in\overline{Q_{T}}}p(x,t)\) in this paper. The initial value matching up to equation (1.1) is

$$ \vert v \vert ^{\beta-1}v(x,0)= \bigl\vert v_{0}(x) \bigr\vert ^{\beta-1}v_{0}(x),\quad x\in\varOmega. $$
(1.2)

While the Dirichlet boundary value condition

$$ v(x,t)=0,\quad (x,t)\in\partial\varOmega\times(0,T) $$
(1.3)

is dispensable.

If \(g=0\), equation (1.1) arises from the branches of flows of electro-rheological or thermo-rheological fluids (see [13]), and the processing of digital images [415]. If the variable exponent \(p(x,t)\) is replaced by a constant p, equation (1.1) becomes the well-known non-Newtonian polytropic filtration equation with orientated convection [16], as well as the convection-diffusion-reaction equation in which the variable can be interpreted as temperature for heat transfer problems, concentration for dispersion problems, etc. [17]. Now, let us give some details in part of the above references. Ye and Yin studied the propagation profile for the equation

$$u_{t}=\operatorname{div}\bigl( \bigl\vert \nabla u^{m} \bigr\vert ^{p-2}\nabla u^{m}\bigr)-\vec{\beta}(x)\cdot \nabla u^{q}, $$

in which the orientation of the convection was specified to be either the convection with counteracting diffusion or the convection with promoting diffusion, that is, \(\vec{\beta}(x)\cdot(-x)\geq0\) or \(\vec{\beta}(x)\cdot x\geq0\), respectively [16]. Guo, Li, and Gao considered the following evolutionary \(p(x)\)-Laplacian equation:

$$v_{t} = \operatorname{div}\bigl( \vert \nabla v \vert ^{p(x)-2} \nabla v\bigr) + \vert v \vert ^{r-2}v,\quad (x,t)\in Q_{T}, $$

subject to homogeneous Dirichlet boundary condition, where \(r > 1\) is a constant. By using the energy estimate method, the regularity of weak solutions and blow-up in finite time were revealed in [7]. Antontsev and Shmarev have published a series of papers [813] on the homogeneous Dirichlet problem for the doubly nonlinear parabolic equation

$$ v_{t} = \operatorname{div}\bigl(b(x,t,v) \vert v \vert ^{\alpha(x,t)} \vert \nabla v \vert ^{p(x,t)-2}\nabla v\bigr) + f(x,t),\quad (x,t)\in Q_{T}, $$

provided that \(a(x,t,v)\geq a>0\). They established conditions on the data that guarantee the comparison principle and uniqueness of bounded weak solutions in suitable Orlicz–Sobolev spaces subject to some additional restrictions [12]. Gao, Chu, and Gao in [14] studied the nonlinear diffusion equation

$$v_{t} = \operatorname{div}\bigl( \vert \nabla v \vert ^{p(x,t)-2} \nabla v+b(x,t)\nabla v\bigr) +f(v),\quad (x,t)\in Q_{T}, $$

with the homogeneous Dirichlet boundary condition (1.3), where f is a continuous function satisfying

$$\bigl\vert f(v) \bigr\vert \leq a_{0} \vert v \vert ^{\alpha-1} , $$

with \(a_{0}>0\) and \(\alpha>1\). They constructed suitable function spaces and used Galerkin’s method to obtain the existence of weak solutions. It is worth pointing out that the requirement on \(p_{t} (x,t)\) is only negative and integrable, which is a weaker condition than the corresponding conditions appearing in other papers. Recently, Liu and Dong [15] generalized [14]’s result to a more general equation

$$v_{t} = \operatorname{div}\bigl( \bigl\vert \nabla v^{m} \bigr\vert ^{p(x,t)-2}\nabla v^{m}+b(x,t)\nabla v^{m}\bigr) +v^{q(x,t)},\quad (x,t)\in Q_{T} $$

and gave a classification of the weak solutions. In addition, the equation arising from the double phase obstacle problems of the type

$$v_{t} = \operatorname{div}\bigl(a(x) \vert \nabla v \vert ^{p-2} \nabla v+b(x) \vert \nabla v \vert ^{q-2}\nabla v\bigr) +f(x,t,v, \nabla v),\quad (x,t)\in Q_{T} $$

has gained a wide attention [18, 19] etc., where \(a(x)+b(x)>0\).

In recent years, we have been interested in the well-posedness of weak solutions to the nonlinear equation

$$ v_{t} = \operatorname{div}\bigl(b(x) \vert \nabla v \vert ^{p(x)-2}\nabla v\bigr) + f(x,t, v, \nabla v),\quad (x,t)\in Q_{T}, $$
(1.4)

with some restrictions in \(f(x,t,v, \nabla v)\). Different from other researchers’ works [715], in which \(b(x)=1\) or \(b(x)>b^{-}>0\), where \(b^{-}=\min_{x\in\overline{\varOmega}}b(x)\), we only assumed that

$$ b(x)>0,\quad x\in\varOmega,\qquad b(x)=0,\quad x\in\partial\varOmega, $$

and proved that the stability of weak solutions may be independent of the Dirichlet boundary value condition (1.3). One might refer to [2024] for the details.

In this paper, for any \(t\in[0,T]\), we assume

$$ b(x,t)>0,\quad x\in\varOmega,\qquad b(x,t)=0,\quad x\in\partial\varOmega. $$
(1.5)

Comparing with equation (1.4), equation (1.1) is with the nonlinearity of \(|v|^{\beta-1}v\), with the diffusion coefficient \(b(x,t)\) and the variable exponent \(p(x,t)\) depending on time variable t, and with a more complicate convection term \(\nabla\vec{g}\cdot \nabla\vec{\gamma}(v)\). These nonlinearities not only bring some essential changes to the proof of the existence, but also add difficulties to proving the stability of weak solutions. The readers will see that, in order to overcome these difficulties, a new technique based on the mean value theorem is posed to prove the uniqueness of weak solution, another new technique based on the proof by contradiction is introduced. Both of them supply a new method to prove uniqueness of weak solution for the nonlinear degenerate parabolic equations.

Definition 1.1

A function \(v(x,t)\) is said to be a weak solution of equation (1.1) with initial value (1.2), provided that \(v(x,t)\) satisfies

$$ \begin{gathered}v \in L_{\mathrm{loc}}^{\infty}\bigl(0,T; W^{1,p(x,t)}( \varOmega)\bigr),\qquad v\in W_{\mathrm{loc}}^{1,2}\bigl((0,T), L^{2}( \varOmega)\bigr),\\ b(x,t) \vert \nabla v \vert ^{p(x,t)} \in L^{1}(Q_{T}),\end{gathered} $$
(1.6)

and for \(\forall\phi(x,t) \in{C_{0}^{1}}(Q_{T})\),

$$ \begin{aligned}[b] & \iint_{Q_{T}} \bigl(- \vert v \vert ^{\beta-1}v \phi_{t}\bigr) \,dx\,dt + \iint_{Q_{T}} b(x,t) \vert \nabla v \vert ^{p(x,t) - 2} \nabla v \cdot\nabla\phi \,dx\,dt \\ &\qquad+ \sum_{i=1}^{N} \iint_{Q_{T}}g^{i}(x,t)\gamma_{i}(v) \phi_{x_{i}}\,dx\,dt \\ &\quad=\sum_{i=1}^{N} \iint_{Q_{T}}\gamma_{i}(v)g^{i}(x,t) \phi(x,t)\,dx\,dt. \end{aligned} $$
(1.7)

Initial value (1.2) is true in the sense

$$ \lim_{t\rightarrow0} \int_{\varOmega} \vert v \vert ^{\beta-1}v(x,t)\varphi(x) \,dx= \int_{\varOmega} \vert v \vert ^{\beta-1}v_{0}(x) \varphi(x) \,dx,\quad\forall\varphi (x)\in C_{0}^{\infty}(\varOmega). $$
(1.8)

The main results are the following theorems.

Theorem 1.2

If\(p^{-}\geq2\), \(b(x,t)\)satisfying (1.5), \(v_{0}(x)\in L^{\infty}(\varOmega)\)is nonnegative for\(i\in\{ 1, 2, \ldots, N\}\), \(\gamma_{i}(s)\)is a\(C^{1}\)function satisfying\(|\gamma_{i}'(s)|^{2}|s|^{1-\beta}\leq c\)for\(i=1,2,\ldots, N\), \(g^{i}(x,t)\)satisfies

$$ \int_{\varOmega}\frac{p(x,t)-2}{p(x,t)} \Biggl(\sum _{i=1}^{N}g^{i}(x,t)b(x,t)^{-\frac{2}{p(x,t)}} \Biggr)^{\frac {p(x,t)}{p(x,t)-2}}\,dx\leq c(T), $$
(1.9)

then there is a nonnegative weak solution of equation (1.1) with initial value (1.2) in the sense of Definition1.1.

Theorem 1.3

Let\(u(x,t)\)and\(v(x,t)\)be two weak solutions of equation (1.1) with the different initial values\(u_{0}(x)\)and\(v_{0}(x)\)respectively, \(0< m\leq\|u\|_{L^{\infty}(Q_{T})}\leq M\), \(0< m\leq\|v\|_{L^{\infty}(Q_{T})}\leq M\). If\(p^{-}>1\), \(\gamma_{i}(s)\)is a Lipschitz function, \(b(x,t)\)satisfies (1.5), and

$$ \Biggl\vert \sum_{i=1}^{N} \frac{\partial g^{i}(x,t)}{\partial x_{i}} \Biggr\vert \leq cb(x,t)^{\frac{\alpha_{1}}{p(x,t)}}, $$
(1.10)

then there exists a constant\(\alpha_{1}\geq2p^{+}\)such that

$$ \begin{aligned}[b] &\int_{\varOmega}b(x,t)^{\frac{\alpha_{1}}{p(x,t)}} \bigl\vert \vert u \vert ^{\beta -1}u(x,t) - \vert v \vert ^{\beta-1}v(x,t) \bigr\vert ^{2}\,dx \\ &\quad\leq \int_{\varOmega}b(x,0)^{\frac{\alpha_{1}}{p(x,0)}} \bigl\vert \vert u_{0} \vert ^{\beta-1}u_{0}(x) - \vert v_{0} \vert ^{\beta-1}v_{0}(x) \bigr\vert ^{2}\,dx,\quad \forall t \in[0,T). \end{aligned} $$
(1.11)

Theorem 1.4

Let\(u(x,t)\)and\(v(x,t)\)be two weak solutions of equation (1.1) with the different initial values\(u_{0}(x)\)and\(v_{0}(x)\)respectively, \(\gamma_{i}(s)\)be a Lipschitz function. Suppose that\(g^{i}(x,t)\)satisfies (1.10) and

$$ \Biggl\vert \sum_{i=1}^{N}g^{i}(x,t) \Biggr\vert \leq cb(x,t)^{\frac{1}{p(x,t)}} $$
(1.12)

and one of the following conditions is true:

  1. (i)

    \(\beta\leq1\).

  2. (ii)

    For\(1\leq i\leq N\), \(\gamma_{i}(s)\)satisfies

    $$ \bigl\vert \gamma_{i}(t_{1})-\gamma_{i}(t_{2}) \bigr\vert \leq c \bigl\vert \vert t_{1} \vert ^{\beta-1}t_{1}- \vert t_{2} \vert ^{\beta -1}t_{2} \bigr\vert . $$
    (1.13)

Then

$$ \begin{aligned}[b] &\int_{\varOmega}\bigl\vert \vert u \vert ^{\beta-1}u(x,t) - \vert v \vert ^{\beta-1}v(x,t) \bigr\vert \,dx \\&\quad\leq \int_{\varOmega}\bigl| \vert u_{0} \vert ^{\beta-1}u_{0}(x) - \vert v_{0} \vert ^{\beta -1}v_{0}(x) \bigr|\,dx,\quad\forall t \in[0,T).\end{aligned} $$
(1.14)

Conditions (1.9), (1.10), and (1.12) all reflect the internal mutually dependent relationships between the diffusion coefficient \(b(x,t)\) and the convective coefficients \(g^{i}(x,t)\). Such an internal mutually dependent relationship that can affect the finite propagation has been studied in [16], while the internal mutually dependent relationships between the diffusion coefficient and the convection term arise in mathematics finance model for studying the agent’s decision under the risk [25].

At the end of introduction, it might be advisable to summarize briefly. First, as a classical work on the well-posed results of the solution of a nonlinear parabolic equation, there are many papers devoted to this problem (one can refer to [2628] and the references therein). Secondly, the model studied in this paper is a parabolic equation with variable exponential term; we would like to point out that more details on the structural characteristics and the physical background of the variable exponential term have been described in [2933], etc. Thirdly, one can see that the new method to prove uniqueness of weak solution can be generalized to study the double phase obstacle problems.

2 The existence of weak solutions

Let us consider the approximate initial-boundary value problem

$$\begin{aligned}& \bigl( \vert v \vert ^{\beta-1}v\bigr)_{t}= \operatorname{div} \bigl(\bigl(b(x,t)+\varepsilon\bigr) \vert \nabla v \vert ^{p(x,t)-2}\nabla v \bigr)+\nabla\vec{g}\cdot\nabla\vec{\gamma}(v), \end{aligned}$$
(2.1)
$$\begin{aligned}& \vert v \vert ^{\beta-1}v(x,0)= \vert v_{0} \vert ^{\beta-1}v_{0}(x), \quad x\in\varOmega, \end{aligned}$$
(2.2)
$$\begin{aligned}& v(x,t)=0,\quad (x,t)\in\partial\varOmega\times(0,T). \end{aligned}$$
(2.3)

Definition 2.1

If \(u(x,t)\) satisfies

$$ v \in L_{\mathrm{loc}}^{\infty}\bigl(0,T; W_{0}^{1,p(x,t)}(\varOmega)\bigr), \qquad v\in W_{\mathrm{loc}}^{1,2} \bigl(0,T; L^{2}(\varOmega)\bigr), $$
(2.4)

and for any \(\phi(x,t) \in{C_{0}^{1}}(Q_{T})\), there holds

$$ \begin{aligned}[b] &{-} \iint_{Q_{T}} \vert v \vert ^{\beta-1}v \phi_{t} \,dx\,dt+ \iint_{Q_{T}} \bigl(b(x,t)+\varepsilon\bigr) \vert \nabla v \vert ^{p(x,t) - 2}\nabla v \cdot\nabla\phi \,dx\,dt \\ &\qquad+ \sum_{i=1}^{N} \iint_{Q_{T}}\gamma_{i}(v)\phi_{x_{i}}\,dx\,dt \\ &\quad=\sum_{i=1}^{N} \iint_{Q_{T}}\gamma_{i}(v)g^{i}(x,t) \phi(x,t)\,dx\,dt. \end{aligned} $$
(2.5)

Then we say that \(v(x,t)\) is said to be the weak solution of problem (2.1)–(2.3).

For any \(k>0\), let

$$a_{k}=k^{2-\beta},\qquad b_{k}=k^{1-\beta} \frac{3-\beta}{2},\quad k=1,2,\ldots. $$

\(\varphi_{k}(v)\) is an even function and is defined as

$$\varphi_{k}(v)=\left \{ \textstyle\begin{array}{l@{\quad}l} \beta v^{\beta-1}, & v\geq k^{-1}, \\ \beta(a_{k}v^{2}+b_{k} v),& 0\leq v< k^{-1}. \end{array}\displaystyle \right . $$

Then \(\varphi_{k}(v)\in C^{1}(\mathbb{R})\), \(\varphi_{k}(v)\rightarrow \beta v^{\beta-1}\) as \(k\rightarrow\infty\). Instead of (2.1)–(2.3), we now consider the following problem:

$$\begin{aligned}& \varphi_{k}(v)v_{t}=\operatorname{div} \biggl( \bigl(b(x,t)+\varepsilon\bigr) \biggl( \vert \nabla v \vert ^{2}+ \frac{1}{k}\biggr)^{\frac{p(x,t)-2}{2}}\nabla v \biggr)+\nabla\vec {g}\cdot\nabla \vec{\gamma}(v), \end{aligned}$$
(2.6)
$$\begin{aligned}& {v_{k}}(x,t) = 0,\quad (x,t) \in\partial\varOmega \times(0,T), \end{aligned}$$
(2.7)
$$\begin{aligned}& v_{k}(x,0) = v_{0k}(x),\quad x\in\varOmega, \end{aligned}$$
(2.8)

where \(\|v_{0k}(x)-v_{0}(x)\|_{p^{+}(0)}\rightarrow0\) as \(k\rightarrow 0\), and \(p^{+}(0)=\max_{x\in\overline{\varOmega}}p(x,0)\). From [814], there is a unique solution \(v_{k\varepsilon}\) of initial-boundary value problem (2.6)–(2.8). Let \(k\rightarrow\infty\). If \(v_{0}(x)\in L^{\infty}(\varOmega)\) is nonnegative, similar to the process subject to the existence of weak solutions in [12](also[34]), one can prove that there is a nonnegative weak solution \(v_{\varepsilon}\in L^{1}(0,T;W_{0}^{1,p(x,t)}(\varOmega))\) to initial-boundary value problem (2.1)–(2.3) in the sense of Definition 2.1. Moreover,

$$ \Vert v_{k\varepsilon} \Vert _{L^{\infty}(Q_{T})}\leq c,\qquad \Vert v_{\varepsilon} \Vert _{L^{\infty}(Q_{T})}\leq c. $$
(2.9)

Proof of Theorem 1.2

Let us choose \(v_{\varepsilon}\) as a test function. Then

$$ \begin{aligned}[b] &\frac{\beta}{\beta+1} \int_{\varOmega}v_{\varepsilon}^{\beta+1}\,dx + \iint_{Q_{T}} \bigl(b(x,t)+\varepsilon\bigr) \vert \nabla v_{\varepsilon} \vert ^{p(x,t)}\,dx\,dt \\ &\qquad- \iint_{Q_{T}} v_{\varepsilon}\nabla\vec{g}\cdot\nabla\vec { \gamma}(v_{\varepsilon}) \,dx\,dt \\ &\quad= \frac{\beta}{\beta+1} \int_{\varOmega}v_{0}(x)^{\beta+1}\,dx. \end{aligned} $$
(2.10)

Since

$$ \begin{aligned}[b] &{-} \iint_{Q_{T}} v_{\varepsilon}\nabla\vec{g}\cdot\nabla\vec { \gamma}(v_{\varepsilon}) \,dx\,dt \\ &\quad=\sum_{i = 1}^{N} \iint_{Q_{T}} v_{\varepsilon}g^{i}(x,t) \frac {\partial\gamma_{i}(v_{\varepsilon})}{\partial x_{i}} \,dx\,dt \\ &\quad=\sum_{i = 1}^{N} \biggl[ \iint_{Q_{T}} v_{\varepsilon}\frac{\partial (\gamma_{i}(v_{\varepsilon})g^{i}(x,t))}{\partial x_{i}} \,dx\,dt- \iint _{Q_{T}} v_{\varepsilon}a_{i}(v_{\varepsilon}) \frac{\partial g^{i}(x,t)}{\partial x_{i}} \,dx\,dt \biggr] \\ &\quad= - \sum_{i = 1}^{N} \iint_{Q_{T}} \frac{\partial v_{\varepsilon}}{\partial x_{i}}g^{i}(x,t) \gamma_{i}(v_{\varepsilon})\,dx\,dt-\sum_{i=1}^{N} \iint_{Q_{T}} v_{\varepsilon}\gamma_{i}(v_{\varepsilon}) \frac {\partial g^{i}(x,t)}{\partial x_{i}} \,dx\,dt \\ &\quad= - \sum_{i = 1}^{N} \int_{\varOmega}\frac{\partial}{\partial x_{i}} \int _{0}^{v_{\varepsilon}} \gamma_{i}(v_{\varepsilon})\,ds\,dx- \sum_{i=1}^{N} \iint _{Q_{T}} v_{\varepsilon}\gamma_{i}(v_{\varepsilon}) \frac{\partial g^{i}(x,t)}{\partial x_{i}} \,dx\,dt \\ &\quad= -\sum_{i=1}^{N} \iint_{Q_{T}} v_{\varepsilon}\gamma_{i}(v_{\varepsilon}) \frac{\partial g^{i}(x,t)}{\partial x_{i}} \,dx\,dt, \end{aligned} $$
(2.11)

we have

$$ \frac{\beta}{\beta+1} \int_{\varOmega}v_{\varepsilon}^{\beta+1}\,dx + \iint_{Q_{T}} \bigl(b(x,t)+\varepsilon\bigr) \vert \nabla v_{\varepsilon} \vert ^{p(x,t)}\,dx\,dt \leqslant c, $$
(2.12)

which implies

$$ \iint_{Q_{T}} b(x,t) \vert \nabla v_{\varepsilon} \vert ^{p(x,t)}\,dx\,dt\leqslant c. $$
(2.13)

Moreover, let us multiply (2.1) with \(v_{\varepsilon t}\), and obtain

$$ \begin{aligned}[b] &\beta \int_{\varOmega}v_{\varepsilon}^{\beta-1}(v_{{\varepsilon}t})^{2}\,dx \\ &\quad= \int_{\varOmega}\operatorname{div}\bigl(\bigl(\rho(x,t)+\varepsilon\bigr) \vert \nabla v_{\varepsilon} \vert ^{p(x,t)-2}\nabla v_{\varepsilon} \bigr)v_{{\varepsilon}t}\,dx+ \int_{\varOmega}\nabla\vec {g}\cdot\nabla\vec{\gamma}(v_{\varepsilon})v_{\varepsilon t}\,dx. \end{aligned} $$
(2.14)

Since

$$ \begin{aligned}[b] & \int_{\varOmega}\operatorname{div}\bigl(\bigl(b(x,t)+\varepsilon\bigr) \vert \nabla v_{\varepsilon} \vert ^{p(x,t)-2}\nabla v_{\varepsilon} \bigr)v_{\varepsilon t}\,dx \\ &\quad=-\frac{1}{2} \int_{\varOmega}\bigl(b(x,t)+\varepsilon\bigr) \vert \nabla v_{\varepsilon} \vert ^{p(x,t)-2} \vert \nabla v_{\varepsilon} \vert ^{2}_{t}\,dx \\ &\quad=-\frac{1}{2} \int_{\varOmega}\bigl(b(x,t)+\varepsilon\bigr)\frac{d}{dt} \int _{0}^{ \vert \nabla v_{\varepsilon} \vert ^{2}}s^{\frac{p(x,t)-2}{2}}\,ds\,dx\\ &\qquad+ \frac{1}{2} \int _{\varOmega}\bigl(b(x,t)+\varepsilon\bigr) \int_{0}^{ \vert \nabla v_{\varepsilon} \vert ^{2}}\frac{d}{dt}s^{\frac{p(x,t)-2}{2}}\,ds\,dx, \end{aligned} $$
(2.15)

and by \(|\gamma_{i}'(s)|^{2}|s|^{1-\beta}\leq c\), \(p^{-}\geq2\) and by (1.9), using the Young inequality, we have

$$ \begin{aligned}[b] & \Biggl\vert \int_{\varOmega}\sum_{i=1}^{N} \frac{\partial \gamma_{i}(v_{\varepsilon})}{\partial x_{i}}g^{i}(x,t)v_{\varepsilon t}\,dx \Biggr\vert \\ &\quad\leq \int_{\varOmega}\sum_{i=1}^{N} \bigl\vert \gamma_{i}'(v_{\varepsilon }) \bigr\vert \bigl\vert g^{i}(x,t)v_{\varepsilon x_{i}} \bigr\vert \vert v_{\varepsilon t} \vert \,dx \\ &\quad\leq\frac{\beta}{2} \int_{\varOmega}v_{\varepsilon}^{\beta -1}(v_{\varepsilon t})^{2}\,dx+ \frac{2}{\beta} \int_{\varOmega}\sum_{i=1}^{N} \bigl\vert \gamma_{i}'(v_{\varepsilon})g^{i}(x,t)v_{\varepsilon x_{i}} \bigr\vert ^{2} \vert v_{\varepsilon} \vert ^{1-\beta} \,dx \\ &\quad\leq\frac{\beta}{2} \int_{\varOmega}v_{\varepsilon}^{\beta -1}(v_{\varepsilon t})^{2}\,dx \\ &\qquad+c \int_{\varOmega} \Biggl[\frac{p(x,t)-2}{p(x,t)} \Biggl(\sum _{i=1}^{N}g^{i}(x,t)b(x,t)^{-\frac{2}{p(x,t)}} \Biggr)^{\frac {p(x,t)}{p(x,t)-2}}+\frac{2}{p(x,t)}b(x,t) \vert \nabla v_{\varepsilon } \vert ^{p(x,t)} \Biggr]\,dx. \end{aligned} $$
(2.16)

From (2.14)–(2.16), we extrapolate that

$$\begin{aligned}& \frac{\beta}{2} \int_{\varOmega}v^{\beta-1}(u_{\varepsilon t})^{2}\,dx+ \frac{1}{2} \int_{\varOmega}\bigl(b(x,t)+\varepsilon\bigr)\frac {d}{dt} \int_{0}^{ \vert \nabla v_{\varepsilon} \vert ^{2}}s^{\frac{p(x,t)-2}{2}}\,ds\,dx \\ & \quad\leq c \int_{\varOmega} \biggl[\frac{p(x,t)-2}{p(x,t)} \bigl(g^{i}(x,t)b(x,t)^{-\frac{2}{p(x,t)}} \bigr)^{\frac {p(x,t)}{p(x,t)-2}}+\frac{2}{p(x,t)}b(x,t) \vert \nabla v_{\varepsilon } \vert ^{p(x,t)} \biggr]\,dx \\& \qquad{}+ \biggl\vert \frac{1}{2} \int_{\varOmega}\bigl(b(x,t)+\varepsilon\bigr) \int _{0}^{ \vert \nabla v_{\varepsilon} \vert ^{2}}\frac{d}{dt}s^{\frac{p(x,t)-2}{2}}\,ds\,dx \biggr\vert \\& \quad\leq c. \end{aligned}$$

Then

$$ \bigl\Vert \bigl(v_{\varepsilon}^{\frac{\beta+1}{2}} \bigr)_{t} \bigr\Vert _{L^{2}(Q_{T})}=\frac{\beta+1}{2} \bigl\Vert v_{\varepsilon}^{\frac{\beta -1}{2}}v_{\varepsilon t} \bigr\Vert _{L^{2}(Q_{T})}\leq c, $$
(2.17)

and

$$ \begin{aligned}[b] \iint_{Q_{T}} \vert v_{\varepsilon t} \vert ^{2}\,dx\,dt&\leq \int_{0}^{T} \int_{\varOmega }v_{\varepsilon}^{\beta-1} \vert v_{\varepsilon t} \vert ^{2}v_{\varepsilon}^{1-\beta}\,dx\,dt\leq \Vert v_{\varepsilon} \Vert _{L^{\infty}(Q_{T})}^{1-\beta } \int_{0}^{T} \int_{\varOmega}v_{\varepsilon}^{\beta-1} \vert v_{\varepsilon t} \vert ^{2}\,dx\,dt\hspace{-24pt}\\&\leq c.\end{aligned} $$
(2.18)

From (2.12), (2.18), we are able to extrapolate that \(v_{\varepsilon}\rightarrow v\) a.e. in \(Q_{T}\). Accordingly, \(\gamma _{i}(v_{\varepsilon})\rightarrow\gamma_{i}(v)\) a.e. in \(Q_{T}\).

Let \(\varepsilon\rightarrow0\) in (2.10). Similar to that in [35], which is subject to the evolutionary p-Laplacian equation, it is not difficult to deduce that

$$\bigl(b(x,t)+\varepsilon\bigr) \vert \nabla v_{\varepsilon} \vert ^{p(x,t)-2}\nabla v_{\varepsilon}\rightharpoonup b(x,t) \vert \nabla v \vert ^{p(x,t)-2}\nabla v, \quad\text{in } L^{1}\bigl(0,T; L^{\frac{p(x,t)}{p(x.t)-1}}(\varOmega)\bigr). $$

Also, we can show that initial value (1.2) is true in the sense of (1.8) as in [12]. Theorem 1.2 is proved. □

3 Proof of Theorem 1.3

Lemma 3.1

([36, 37])

  1. (i)

    The space\((L^{p(x)}(\varOmega), \|\cdot\| _{L^{p(x)}(\varOmega)} )\), \((W^{1,p(x)}(\varOmega), \|\cdot\|_{W^{1,p(x)}(\varOmega)} )\)and\(W^{1,p(x)}_{0}(\varOmega)\)are reflexive Banach spaces.

  2. (ii)

    Let\(p(x)\)and\(q(x)\)be two functions with\(\frac{1}{p(x)}+\frac{1}{q(x)} = 1\). The conjugate space of\(L^{p(x)}(\varOmega)\)is\(L^{q(x)}(\varOmega)\). For any\(u \in L^{p(x)}(\varOmega)\)and\(v \in L^{q(x)}(\varOmega)\),

    $$\biggl\vert \int_{\varOmega}uv \,dx \biggr\vert \leq2 \Vert u \Vert _{L^{p(x)}(\varOmega)} \Vert v \Vert _{L^{q(x)}(\varOmega)}. $$
  3. (iii)
    $$ \begin{gathered} \textit{If } \Vert u \Vert _{L^{p(x)}(\varOmega)} = 1,\quad \textit{then } \int_{\varOmega} \vert u \vert ^{p(x)} \,dx = 1. \\ \textit{If } \Vert u \Vert _{L^{p(x)}(\varOmega) }> 1,\quad \textit{then } \vert u \vert ^{p^{-}}_{L^{p(x)}(\varOmega ) }\leq \int_{\varOmega} \vert u \vert ^{p(x)} \,dx\leq \vert u \vert ^{p^{+}}_{L^{p(x)}(\varOmega) }. \\ \textit{If } \Vert u \Vert _{L^{p(x)}(\varOmega)} < 1,\quad \textit{then } \vert u \vert ^{p^{+}}_{L^{p(x)}(\varOmega ) }\leq \int_{\varOmega} \vert u \vert ^{p(x)} \,dx\leq \vert u \vert ^{p^{-}}_{L^{p(x)}(\varOmega) }. \end{gathered} $$

Proof of Theorem 1.3

For any given \(t\in(0,T)\) and small enough \(\lambda>0\), we denote \(\varOmega_{\lambda t}=\{x\in\varOmega: b(x,t)>\lambda\}\) and define

$$\xi_{\lambda}(x,t)=\bigl[b(x,t)-\lambda\bigr]_{+}^{\frac{\alpha_{1}}{p(x,t)}}, $$

where \(\alpha_{1}\geq2p^{+}\).

We choose \(\chi_{[\tau,s]}(t)[u(x,t)-v(x,t)]\xi_{\lambda}(x,t)\) as a test function, where \(\chi_{[\tau,s]}\) is the characteristic function on \([\tau,s]\subset(0,t)\). Then

$$ \begin{aligned}[b] & \iint_{Q_{\tau s}}(u-v)\xi_{\lambda}(x,t)\frac{\partial ( \vert u \vert ^{\beta-1}u - \vert v \vert ^{\beta-1}v)}{\partial t}\,dx\,dt \\ &\quad=- \iint_{Q_{\tau s}} \bigl(b(x,t) \vert \nabla u \vert ^{p(x,t)-2}\nabla u- \vert \nabla v \vert ^{p(x,t)-2}\nabla v \bigr) \nabla\bigl[(u-v)\xi_{\lambda }(x,t)\bigr] \,dx\,dt \\ &\qquad-\sum_{i=1}^{N} \iint_{Q_{\tau s}}g^{i}(x,t)\bigl[\gamma_{i}(u)- \gamma _{i}(v)\bigr] \bigl[u-v)\xi_{\lambda}(x,t) \bigr]_{x_{i}} \,dx\,dt \\ &\qquad-\sum_{i=1}^{N} \iint_{Q_{\tau s}}\bigl[\gamma_{i}(u)-\gamma _{i}(v)\bigr] \bigl[(u-v)\xi_{\lambda}(x,t)\bigr] \frac{\partial g^{i}(x,t)}{\partial x_{i}}\,dx\,dt, \end{aligned} $$
(3.1)

where \(Q_{\tau s}=\varOmega\times[\tau, s]\) as usual.

In the first place,

$$ \begin{aligned}[b] & \iint_{Q_{\tau s}}b(x,t) \bigl( \vert \nabla u \vert ^{p(x,t)-2}\nabla u- \vert \nabla v \vert ^{p(x,t)-2}\nabla v \bigr) \nabla \bigl[(u-v)\xi_{\lambda} \bigr] \,dx\,dt \\ &\quad= \iint_{Q_{\tau s}}b(x,t)\xi_{\lambda} \bigl( \vert \nabla u \vert ^{p(x,t)-2}\nabla u- \vert \nabla v \vert ^{p(x,t)-2}\nabla v \bigr)\nabla(u-v) \,dx\,dt \\ &\qquad+ \iint_{Q_{\tau s}}b(x,t) \bigl( \vert \nabla u \vert ^{p(x,t)-2}\nabla u- \vert \nabla v \vert ^{p(x,t)-2}\nabla v \bigr) (u-v)\nabla\xi_{\lambda} \,dx\,dt, \end{aligned} $$
(3.2)

we have

$$ \iint_{Q_{\tau s}}b(x,t)\xi_{\lambda} \bigl( \vert \nabla u \vert ^{p(x,t)-2}\nabla u- \vert \nabla v \vert ^{p(x,t)-2}\nabla v \bigr)\nabla(u-v) \,dx\,dt\geq0 $$
(3.3)

and

$$ \begin{aligned}[b] & \biggl\vert \iint_{Q_{\tau s}}(u-v)b(x,t) \bigl( \vert \nabla u \vert ^{p(x,t)-2}\nabla u- \vert \nabla v \vert ^{p(x,t)-2}\nabla v \bigr) \nabla\xi _{\lambda}\,dx\,dt \biggr\vert \\ &\quad\leq c \biggl( \int_{\tau}^{s} \int_{\varOmega_{\lambda t}}b(x,t) \bigl( \vert \nabla u \vert ^{p(x,t)}+ \vert \nabla v \vert ^{p(x,t)} \bigr)\,dx\,dt \biggr)^{\frac{1}{q_{1}}} \\ &\qquad\cdot \biggl( \int_{\tau}^{s} \int_{\varOmega_{\lambda t}}b(x,t) \vert \nabla\xi_{\lambda} \vert ^{p(x,t)} \vert u-v \vert ^{p(x,t)}\,dx\,dt \biggr)^{\frac{1}{p_{1}}} \\ &\quad\leq c \biggl( \int_{\tau}^{s} \int_{\varOmega}b(x,t) \bigl( \vert \nabla u \vert ^{p(x,t)}+ \vert \nabla v \vert ^{p(x,t)} \bigr)\,dx\,dt \biggr)^{\frac{1}{q_{1}}} \\ &\qquad\cdot \biggl( \int_{\tau}^{s} \int_{\varOmega_{\lambda t}}b(x,t)\bigl[b(x,t)-\lambda\bigr]^{p(x,t) (\frac{\alpha _{1}}{p(x,t)}-1 )} \vert \nabla b \vert ^{p(x,t)} \vert u-v \vert ^{p(x,t)}\,dx\,dt \biggr)^{\frac{1}{p_{1}}} \\ &\quad\leq c \biggl( \int_{\tau}^{s} \int_{\varOmega_{\lambda t}}b(x,t)\bigl[b(x,t)-\lambda\bigr]^{p(x,t) (\frac{\alpha _{1}}{p(x,t)}-1 )} \vert u-v \vert ^{p(x,t)}\,dx\,dt \biggr)^{\frac{1}{p_{1}}}. \end{aligned} $$
(3.4)

Here, \(q(x,t)=\frac{p(x,t)}{p(x,t)-1}\), from (iii) of Lemma 3.1, \(q_{1}=q^{+}\) or \(q^{-}\) according to

$$\int_{\tau}^{s} \int_{\varOmega_{\lambda t}}b(x,t) \bigl( \vert \nabla u \vert ^{p(x,t)}+ \vert \nabla v \vert ^{p(x,t)} \bigr)\,dx\,dt< 1, $$

or

$$\int_{\tau}^{s} \int_{\varOmega_{\lambda t}}b(x,t) \bigl( \vert \nabla u \vert ^{p(x,t)}+ \vert \nabla v \vert ^{p(x,t)} \bigr)\,dx\,dt\geq1, $$

\(p_{1}\) has a similar meaning, and we have used the fact that \(|\nabla b|\leq c\) in (3.4).

Then

$$ \begin{aligned}[b] &\lim_{\lambda\rightarrow0} \biggl\vert \iint_{Q_{\tau s}}(u-v)b(x,t) \bigl( \vert \nabla u \vert ^{p(x,t)-2}\nabla u- \vert \nabla v \vert ^{p(x,t)-2}\nabla v \bigr) \nabla\xi_{\lambda}\,dx\,dt \biggr\vert \\ &\quad\leq\lim_{\lambda\rightarrow0}c \biggl( \int_{\tau}^{s} \int _{\varOmega_{\lambda t}}b(x,t) \bigl(b(x,t)-\lambda\bigr)^{p(x,t)(\frac{\alpha _{1}}{p(x,t)}-1)} \vert u-v \vert ^{p(x,t)}\,dx\,dt \biggr)^{\frac{1}{p_{1}}} \\ &\quad\leq c \biggl( \int_{\tau}^{s} \int_{\varOmega}b(x,t)^{1+p(x,t) (\frac{\alpha_{1}}{p(x,t)}-1 )} \vert u-v \vert ^{p(x,t)}\,dx\,dt \biggr)^{\frac {1}{p_{1}}}. \end{aligned} $$
(3.5)

If we denote that

$$\varOmega_{1t}=\bigl\{ x\in\varOmega: p(x,t)\geq2\bigr\} ,\qquad \varOmega_{2t}=\bigl\{ x\in\varOmega : p(x,t)< 2\bigr\} , $$

by \(u,v\in L^{\infty}\), we have

$$ \begin{aligned}[b] & \biggl( \int_{\tau}^{s} \int_{\varOmega_{1t}}b(x,t)^{1+p(x,t) (\frac{\alpha_{1}}{p(x,t)}-1 )} \vert u-v \vert ^{p(x,t)}\,dx\,dt \biggr)^{\frac{1}{p_{1}}} \\ &\quad\leq c \biggl( \iint_{Q_{\tau s}}b(x,t)^{\frac{\alpha _{1}}{p(x,t)}} \vert u-v \vert ^{2}\,dx\,dt \biggr)^{\frac{1}{p_{1}}}, \end{aligned} $$
(3.6)

and using the Hölder inequality, we get

$$ \begin{aligned} [b]& \biggl( \int_{\tau}^{s} \int_{\varOmega_{2t}}b(x,t)^{1+p(x,t) (\frac{\alpha_{1}}{p(x,t)}-1 )} \vert u-v \vert ^{p(x,t)}\,dx\,dt \biggr)^{\frac{1}{p_{1}}} \\ &\quad\leq \biggl( \int_{\tau}^{s} \int_{\varOmega_{2t}} \bigl[b(x,t)^{1+p(x,t) (\frac{\alpha_{1}}{p(x,t)}-1)-\frac{\alpha_{1}}{2}} \bigr]^{\frac {2}{2-p(x,t)}}\,dx\,dt \biggr)^{\frac{1}{p_{12}}\frac{1}{p_{1}}} \\ &\qquad\cdot \biggl( \int_{\tau}^{s} \int_{\varOmega_{2t}}b(x,t)^{\frac {\alpha_{1}}{p(x,t)}} \vert u-v \vert ^{2}\,dx\,dt \biggr)^{\frac{1}{2}} \\ &\quad\leq c \biggl( \iint_{Q_{\tau s}}b(x,t)^{\frac{\alpha _{1}}{p(x,t)}} \vert u-v \vert ^{2}\,dx\,dt \biggr)^{\frac{1}{2}}, \end{aligned} $$
(3.7)

where \(p_{12}(x,t)=\frac{2}{2-p(x,t)}\), from (iii) of Lemma 3.1, \(p_{12}=p_{12}^{+}\) or \(p_{12}^{-}\) according to

$$\int_{\tau}^{s} \int_{\varOmega_{2t}} \bigl[b(x,t)^{1+p(x,t) (\frac{\alpha_{1}}{p(x,t)}-1)-\frac{\alpha_{1}}{2}} \bigr]^{\frac {2}{2-p(x,t)}}\,dx\,dt< 1, $$

or

$$\int_{\tau}^{s} \int_{\varOmega_{2t}} \bigl[b(x,t)^{1+p(x,t) (\frac{\alpha_{1}}{p(x,t)}-1)-\frac{\alpha_{1}}{2}} \bigr]^{\frac {2}{2-p(x,t)}}\,dx\,dt \geq1. $$

In the second place,

$$ \begin{aligned}[b] &\sum_{i=1}^{N} \iint_{Q_{\tau s}}\bigl[\gamma_{i}(u)-\gamma _{i}(v)\bigr] \bigl[(u-v)\xi_{\lambda}\bigr]_{x_{i}} \,dx\,dt \\ &\quad=\sum_{i=1}^{N} \iint_{Q_{\tau s}}\bigl[\gamma_{i}(u)-\gamma _{i}(v)\bigr](u-v)\xi_{\lambda x_{i}} \,dx\,dt \\ &\qquad+\sum_{i=1}^{N} \iint_{Q_{\tau s}}\bigl[\gamma_{i}(u)-\gamma _{i}(v)\bigr](u-v)_{x_{i}}\xi_{\lambda} \,dx\,dt. \end{aligned} $$
(3.8)

Since \(|\nabla b|\leq c\), \(\alpha_{1}\geq2p^{+}\), there hold

$$ \begin{aligned}[b] &\lim_{\lambda\rightarrow0}\sum _{i=1}^{N} \iint_{Q_{\tau s}}\bigl[\gamma _{i}(u)- \gamma_{i}(v)\bigr](u-v)\xi_{\lambda x_{i}} \,dx\,dt \\ &\quad\leq c\lim_{\lambda\rightarrow0}\sum_{i=1}^{N} \int_{\tau}^{s} \int _{\varOmega_{\lambda t}}\bigl[\gamma_{i}(u)-\gamma _{i}(v)\bigr](u-v)\bigl[b(x,t)-\lambda\bigr]^{\frac{\alpha_{1}}{p(x,t)}-1} \vert b_{ x_{i}} \vert \,dx \\ &\quad\leq c \biggl( \iint_{Q_{\tau s}}b(x,t)^{\frac{\alpha _{1}}{p(x,t)}} \vert u-v \vert ^{2}\,dx \biggr)^{\frac{1}{2}} \end{aligned} $$
(3.9)

and

$$ \begin{aligned} [b]&\lim_{\lambda\rightarrow0}\sum _{i=1}^{N} \iint_{Q_{\tau s}}\bigl[\gamma _{i}(u)- \gamma_{i}(v)\bigr](u-v)_{x_{i}}\xi_{\lambda} \,dx\,dt \\ &\quad=\lim_{\lambda\rightarrow0}\sum_{i=1}^{N} \iint_{Q_{\tau s}}\bigl[\gamma _{i}(u)- \gamma_{i}(v)\bigr](u-v)_{x_{i}}\bigl[b(x,t)-\lambda \bigr]_{+}^{\frac{\alpha _{1}}{p(x,t)}} \,dx\,dt \\ &\quad\leq\sum_{i=1}^{N} \biggl( \iint_{Q_{\tau s}} \bigl(b(x,t)^{\frac {\alpha_{1}-1}{p(x,t)}} \bigr)^{q(x,t)} \bigl\vert \gamma_{i}(u)-\gamma _{i}(v) \bigr\vert ^{p'(x,t)}\,dx\,dt \biggr)^{\frac{1}{q_{1}}} \\ &\qquad\cdot \biggl( \iint_{Q_{\tau s}}b(x,t) \bigl( \vert \nabla u \vert ^{p(x,t)}+ \vert \nabla v \vert ^{p(x,t)} \bigr) \,dx\,dt \biggr)^{\frac{1}{p_{1}}} \\ &\quad\leq c\sum_{i=1}^{N} \biggl( \iint_{Q_{\tau s}}b(x,t)^{\frac{\alpha _{1}-1}{p(x,t)-1}} \bigl\vert \gamma_{i}(u)-\gamma_{i}(v) \bigr\vert ^{q(x,t)}\,dx\,dt \biggr)^{\frac{1}{q_{1}}}. \end{aligned} $$
(3.10)

When \(1< p(x,t)<2\), we know \(q(x,t)>2\). Since \(\alpha_{1}\geq p^{+}\), if \(b(x,t)<1\), then \(b(x,t)^{\frac{\alpha_{1}-1}{p(x,t)-1}}\leq b(x,t)^{\frac{\alpha_{1}}{p(x,t)}}\). If \(1\leq b(x,t)\leq D=\max_{(x,t)\in\overline{\varOmega}\times[0,T]}b(x,t)\), then

$$b(x,t)^{\frac{\alpha_{1}-1}{p(x,t)-1}-\frac{\alpha _{1}}{p(x,t)}}=b(x,t)^{\frac{\alpha_{1}-p(x,t)}{p(x,t)(p(x,t)-1)}}\leq D^{\frac{\alpha_{1}-p(x,t)}{p(x,t)(p(x,t)-1)}}\leq c, $$

which implies that \(b(x,t)^{\frac{\alpha_{1}-1}{p(x,t)-1}}\leq cb(x,t)^{\frac{\alpha_{1}}{p(x,t)}}\) is always true. Thus, we extrapolate that

$$ \begin{aligned}[b] &\sum_{i=1}^{N} \biggl( \int_{\tau}^{s} \int_{\varOmega_{1t}}b(x,t)^{\frac {\alpha_{1}-1}{p(x,t)-1}} \bigl\vert \gamma_{i}(u)-\gamma _{i}(v) \bigr\vert ^{q(x,t)}\,dx\,dt \biggr)^{\frac{1}{q_{1}}} \\ &\quad\leq c \biggl( \iint_{Q_{\tau s}}\rho(x,t)^{\frac{\alpha _{1}}{p(x,t)}} \vert u-v \vert ^{2}\,dx\,dt \biggr)^{\frac{1}{q_{1}}}. \end{aligned} $$
(3.11)

When \(p(x,t)\geq2\), we know \(q(x,t)<2\). By \(\alpha_{1}\geq2\), using the Hölder inequality, we have

$$ \begin{aligned}[b] &\sum_{i=1}^{N} \biggl( \int_{\tau}^{s} \int_{\varOmega_{2t}}b(x,t)^{\frac {\alpha_{1}-1}{p(x,t)-1}} \bigl\vert \gamma_{i}(u)-\gamma _{i}(v) \bigr\vert ^{p'(x,t)}\,dx\,dt \biggr)^{\frac{1}{q_{1}}} \\ &\quad\leq c \biggl( \iint_{Q_{\tau s}} \bigl[b(x,t)^{\frac{\alpha_{1}-1}{p(x,t)-1}-\frac{\alpha _{1}}{2(p(x,t)-1)}} \bigr]^{\frac{2}{2-q(x,t)}}\,dx\,dt \biggr)^{\frac {1}{q_{22}}\frac{1}{q_{1}}} \\ &\qquad\cdot \biggl( \iint_{Q_{\tau s}}b(x,t)^{\frac{\alpha _{1}}{p(x,t)}} \vert u-v \vert ^{2}\,dx\,dt \biggr)^{\frac{1}{2}} \\ &\quad\leq c \biggl( \iint_{Q_{\tau s}}b(x,t)^{\frac{\alpha _{1}}{p(x,t)}} \vert u-v \vert ^{2}\,dx\,dt \biggr)^{\frac{1}{2}}, \end{aligned} $$
(3.12)

where \(q_{22}(x,t)=\frac{2-q(x,t)}{2}\), \(q_{22}={q}_{22}^{+}\), or \({q}_{22}^{-}\).

In the third place, since \(\vert \sum_{i=1}^{N}\frac{\partial g^{i}(x,t)}{\partial x_{i}} \vert \leq cb(x,t)^{\frac{\alpha_{1}}{p(x,t)}}\)

$$ \begin{aligned}[b] & \Biggl\vert -\lim _{\lambda\rightarrow0}\sum_{i=1}^{N} \iint_{Q_{\tau s}}\bigl[\gamma_{i}(u)-\gamma_{i}(v) \bigr] \bigl[(u-v)\xi_{\lambda}(x,t)\bigr]\frac {\partial g^{i}(x,t)}{\partial x_{i}}\,dx\,dt \Biggr\vert \\ &\quad= \Biggl\vert -\sum_{i=1}^{N} \iint_{Q_{\tau s}}\bigl[\gamma_{i}(u)-\gamma _{i}(v)\bigr](u-v)\frac{\partial g^{i}(x,t)}{\partial x_{i}}\,dx\,dt \Biggr\vert \\ &\quad\leq c \iint_{Q_{\tau s}} \vert u-v \vert ^{2} \Biggl\vert \sum_{i=1}^{N}\frac{\partial g^{i}(x,t)}{\partial x_{i}} \Biggr\vert \,dx\,dt \\ &\quad\leq c \iint_{Q_{\tau s}}b(x,t)^{\frac{\alpha_{1}}{p(x,t)}} \vert u-v \vert ^{2}\,dx\,dt. \end{aligned} $$
(3.13)

From (3.4)–(3.13), letting \(\lambda\rightarrow0\) in (3.1), we deduce that

$$ \begin{aligned}[b] & \iint_{Q_{\tau s}}b(x,t)^{\frac{\alpha_{1}}{p(x,t)}}(u-v)\frac {\partial ( \vert u \vert ^{\beta-1}u - \vert v \vert ^{\beta-1}v )}{\partial t}\,dx\,dt \\ &\quad\leq c \biggl( \iint_{Q_{\tau s}}b(x,t)^{\frac{\alpha _{1}}{p(x,t)}} \bigl\vert u(x,t)-v(x,t) \bigr\vert ^{2}\,dx\,dt \biggr)^{l}, \end{aligned} $$
(3.14)

where \(l\leq1\).

Last but not least, by the mean value theorem,

$$ \begin{aligned}[b] & \iint_{Q_{\tau s}}b(x,t)^{\frac{\alpha_{1}}{p(x,t)}}(u-v)\frac {\partial ( \vert u \vert ^{\beta-1}u - \vert v \vert ^{\beta-1}v )}{\partial t}\,dx\,dt \\ &\quad= \iint_{Q_{\tau s}}b(x,t)^{\frac{\alpha_{1}}{p(x,t)}}\frac {u-v}{ \vert u \vert ^{\beta-1}u - \vert v \vert ^{\beta-1}v} \bigl( \vert u \vert ^{\beta-1}u - \vert v \vert ^{\beta-1}v \bigr) \frac{\partial( \vert u \vert ^{\beta-1}u - \vert v \vert ^{\beta -1}v)}{\partial t}\,dx\,dt\hspace{-18pt} \\ &\quad=\frac{1}{2} \iint_{Q_{\tau s}}\frac{b(x,t)^{\frac{\alpha _{1}}{p(x,t)}}}{\beta \vert \zeta \vert ^{\beta-1}}\frac{\partial ( \vert u \vert ^{\beta-1}u - \vert v \vert ^{\beta-1}v )^{2}}{\partial t}\,dx\,dt, \end{aligned} $$
(3.15)

where \(\zeta\in(v,u)\).

One of the possibilities of (3.15) is that, for any \(s\geq \tau\),

$$ \frac{d}{dt} \bigl\Vert b(x,t)^{\frac{\alpha_{1}}{2p(x,t)}}\bigl( \vert u \vert ^{\beta -1}u - \vert v \vert ^{\beta-1}v\bigr) \bigr\Vert _{L^{2}(\varOmega)}\leq0,\quad t\in[\tau,s], $$
(3.16)

is true, then

$$\begin{gathered} \int_{\varOmega}b(x,t)^{\frac{\alpha_{1}}{p(x,t)}} \bigl\vert \vert u \vert ^{\beta -1}u(x,s) - \vert v \vert ^{\beta-1}v(x,s) \bigr\vert ^{2}\,dx\\\quad\leq \int_{\varOmega}b(x,t)^{\frac{\alpha_{1}}{p(x,t)}} \bigl\vert \vert u \vert ^{\beta-1}u(x,\tau) - \vert v \vert ^{\beta-1}v(x, \tau) \bigr\vert ^{2}\,dx\end{gathered} $$

is clear.

Another possibility of (3.15) is that there is \(s_{0}\geq\tau\) such that

$$ \frac{d}{dt} \bigl\Vert b(x,t)^{\frac{\alpha_{1}}{2p(x,t)}} \bigl( \vert u \vert ^{\beta-1}u - \vert v \vert ^{\beta-1}v \bigr) \bigr\Vert _{L^{2}(\varOmega)}> 0,\quad t\in[\tau,s_{0}], $$
(3.17)

then

$$ \begin{aligned}[b] & \iint_{Q_{\tau s_{0}}}b(x,t)^{\frac{\alpha_{1}}{p(x,t)}}(u-v)\frac {\partial ( \vert u \vert ^{\beta-1}u - \vert v \vert ^{\beta-1}v )}{\partial t}\,dx\,dt \\ &\quad=\frac{1}{2} \iint_{Q_{\tau s_{0}}}\frac{b(x,t)^{\frac{\alpha _{1}}{p(x,t)}}}{\beta \vert \zeta \vert ^{\beta-1}}\frac{\partial ( \vert u \vert ^{\beta-1}u - \vert v \vert ^{\beta-1}v )^{2}}{\partial t}\,dx\,dt \\ &\quad\geq\frac{1}{2\beta M^{\beta-1}} \iint_{Q_{\tau s_{0}}}b(x,t)^{\frac {\alpha_{1}}{p(x,t)}}\frac{\partial ( \vert u \vert ^{\beta-1}u - \vert v \vert ^{\beta -1}v )^{2}}{\partial t}\,dx\,dt, \end{aligned} $$
(3.18)

where \(\zeta\in(v,u)\), \(M=\max \{\|u\|_{L^{\infty}(Q_{T})}, \|v\| _{L^{\infty}(Q_{T})} \}\).

Combining (3.14)–(3.15) with (3.18), we can extrapolate that

$$ \begin{aligned}[b] & \int_{\varOmega}b(x,s_{0})^{\frac{\alpha_{1}}{p(x,s_{0})}} \bigl\vert \vert u \vert ^{\beta-1}u(x,s_{0}) - \vert v \vert ^{\beta-1}v(x,s_{0}) \bigr\vert ^{2}\,dx \\ &\quad\leq \int_{\varOmega}b(x,\tau)^{\frac{\alpha_{1}}{p(x,\tau)}} \bigl\vert \vert u \vert ^{\beta-1}u(x,\tau) - \vert v \vert ^{\beta-1}v(x,\tau) \bigr\vert ^{2}\,dx \\ &\qquad+\frac{2 cM^{\beta-1}}{\beta m^{2(\beta-1)}} \biggl( \int_{\tau}^{s_{0}} \int_{\varOmega}b(x,t)^{\frac{\alpha_{1}}{p(x,t)}} \bigl\vert \vert u \vert ^{\beta -1}u(x,t) - \vert v \vert ^{\beta-1}v(x,t) \bigr\vert ^{2}\,dx\,dt \biggr)^{l}. \end{aligned} $$
(3.19)

Here \(m=\min \{\|u\|_{L^{\infty}(Q_{T})}, \|v\|_{L^{\infty }(Q_{T})} \}\). From (3.19), we have

$$\begin{gathered} \int_{\varOmega}b(x,s_{0})^{\frac{\alpha_{1}}{p(x,s_{0}}} \bigl\vert \vert u \vert ^{\beta -1}u(x,s_{0}) - \vert v \vert ^{\beta-1}v(x,s_{0}) \bigr\vert ^{2}\,dx \\ \quad\leq \int_{\varOmega}b(x,\tau)^{\frac{\alpha_{1}}{p(x,\tau)}} \bigl\vert \vert u \vert ^{\beta-1}u(x,\tau) - \vert v \vert ^{\beta-1}v(x,\tau) \bigr\vert ^{2}\,dx, \end{gathered} $$

which contradicts assumption (3.17). In other words, (3.17) is impossible. This fact implies that, for any \(s,\tau\in [0,T)\), inequality (3.16) is always true. By the arbitrariness of τ, we have

$$ \begin{aligned} & \int_{\varOmega}b(x,s)^{\frac{\alpha_{1}}{p(x,s)}} \bigl\vert \vert u \vert ^{\beta -1}u(x,s)- \vert v \vert ^{\beta-1}v(x,s) \bigr\vert ^{2} \,dx \\ &\quad\leq \int_{\varOmega}b(x,0)^{\frac{\alpha_{1}}{p(x,0)}} \bigl\vert \vert u_{0} \vert ^{\beta-1}u_{0}(x) - \vert v_{0} \vert ^{\beta-1}v_{0}(x) \bigr\vert ^{2}\,dx, \end{aligned} $$

Theorem 1.3 follows. □

4 The stability of weak solutions

Let \({h_{n}}(u)\) be an odd function defined as

$$h_{n}(u) = \left \{ \textstyle\begin{array}{l@{\quad}l} 1, & u > \frac{1}{n}, \\ n^{2}u^{2}e^{1 - n^{2}u^{2}},& 0\leq u \leqslant\frac{1}{n}. \end{array}\displaystyle \right . $$

Then

$$\begin{aligned}& \lim_{n\rightarrow\infty}h_{n}(u)=\operatorname{sign}(u),\quad u\in(-\infty, +\infty), \end{aligned}$$
(4.1)
$$\begin{aligned}& 0\leq h_{n}'(u)\leq\frac{c}{u},\qquad 0< u< \frac{1}{n}, \qquad\lim_{n\rightarrow \infty}h'_{n}(u)u=0. \end{aligned}$$
(4.2)

Proof of Theorem 1.4

Since \(g^{i}(x,t)\) satisfies (1.10), then from Theorem 1.3 we know that the weak solution of equation (1.1) with initial value (1.2) is unique. Let \(u(x,t)\) and \(v(x,t)\) be two solutions of equation (1.1) with the different initial values \(u_{0}(x)\) and \(v_{0}(x)\) respectively. Since the weak solution of equation (1.1) with initial value (1.2) is unique, there are two asymptotic solutions of asymptotic problem (2.1)–(2.3), \(u_{\varepsilon}\) and \(v_{\varepsilon}\), satisfying

$$ \lim_{\varepsilon\rightarrow0}u_{\varepsilon}=u,\qquad \lim _{\varepsilon \rightarrow0}v_{\varepsilon}=v,\quad \text{a.e. }(x,t)\in Q_{T}, $$
(4.3)

and

$$ \begin{gathered} b(x,t)^{\frac{1}{p(x,t)}}\nabla u_{\varepsilon}\rightharpoonup b(x,t)^{\frac{1}{p(x,t)}}\nabla u,\\ b(x,t)^{\frac{1}{p(x,t)}}\nabla v_{\varepsilon}\rightharpoonup b(x,t)^{\frac{1}{p(x,t)}}\nabla v,\quad \text{in } L^{1} \bigl(0,T; L^{p(x,t)}(\varOmega)\bigr).\end{gathered} $$
(4.4)

We now choose \(\chi_{[\tau,s]}(t){h_{n}}(u_{\varepsilon}(x,t) - v_{\varepsilon}(x,t))\) as a test function, and so

$$ \begin{aligned}[b] & \iint_{Q_{\tau s}} h_{n}(u_{\varepsilon} - v_{\varepsilon})\frac {\partial ( \vert u \vert ^{\beta-1}u - \vert v \vert ^{\beta-1}v )}{\partial t}\,dx\,dt \\ &\qquad+ \iint_{Q_{\tau s}}b(x,t) \bigl( \vert \nabla u \vert ^{p(x,t)- 2}\nabla u - \vert \nabla v \vert ^{p(x,t)- 2}\nabla v \bigr) \cdot\nabla(u_{\varepsilon}- v_{\varepsilon})h'_{n}(u_{\varepsilon} - v_{\varepsilon}) \,dx\,dt \\ &\qquad+ \sum_{i = 1}^{N} \iint_{Q_{\tau s}}g^{i}(x,t)\bigl[\gamma_{i}(u) - \gamma _{i}(v)\bigr] \cdot(u_{\varepsilon} - v_{\varepsilon})_{x_{i}}h'_{n}(u_{\varepsilon} - v_{\varepsilon}) \,dx\,dt \\ &\quad=-\sum_{i = 1}^{N} \iint_{Q_{\tau s}} \bigl[\gamma_{i}(u) - \gamma _{i}(v)\bigr]h_{n}(u_{\varepsilon} - v_{\varepsilon}) \frac{\partial g^{i}(x,t)}{\partial x_{i}} \,dx\,dt. \end{aligned} $$
(4.5)

In the first place, (4.4) yields

$$ \begin{aligned}[b] &\lim_{\varepsilon\rightarrow0} \iint_{Q_{\tau s}} \bigl[b(x,t)^{\frac{p(x,t)-1}{p(x,t)}} \bigl( \vert \nabla u \vert ^{p(x,t)-2}\nabla u- \vert \nabla v \vert ^{p(x,t)-2}\nabla v \bigr) \bigr] \\ &\qquad\cdot b(x,t)^{\frac{1}{p(x,t)}}\bigl[\nabla(u_{\varepsilon }-v_{\varepsilon})- \nabla(u-v)\bigr] \,dx\,dt \\ &\quad=0. \end{aligned} $$
(4.6)

In the second place, by (4.6) and the second mean value theorem, we have

$$ \begin{aligned}[b] &\lim_{\varepsilon\rightarrow0} \iint_{Q_{\tau s}}b(x,t)^{\frac {p(x,t)-1}{p(x,t)}} \bigl( \vert \nabla u \vert ^{p(x,t)-2}\nabla u- \vert \nabla v \vert ^{p(x,t)-2}\nabla v \bigr) \\ &\qquad\cdot b(x,t)^{\frac{1}{p(x,t)}}\bigl[\nabla(u_{\varepsilon }-v_{\varepsilon})- \nabla(u-v)\bigr]h'_{n}(u-v) \,dx\,dt \\ &\quad=h'_{n}(\zeta)\lim_{\varepsilon\rightarrow0} \iint_{Q_{\tau s}}b(x,t)^{\frac{p(x,t)-1}{p(x,t)}} \bigl( \vert \nabla u \vert ^{p(x,t)-2}\nabla u- \vert \nabla v \vert ^{p(x,t)-2}\nabla v \bigr) \\ &\qquad\cdot b(x,t)^{\frac{1}{p(x,t)}}\bigl[\nabla(u_{\varepsilon }-v_{\varepsilon})- \nabla(u-v)\bigr]\,dx\,dt \\ &\quad=0, \end{aligned} $$
(4.7)

and since \((u_{\varepsilon}-v_{\varepsilon})\rightarrow(u-v)\), a.e. in Ω,

$$\bigl\vert \bigl[h'_{n}(u_{\varepsilon}-v_{\varepsilon})-h'_{n}(u-v) \bigr] \bigr\vert \leq c(n), $$

by (4.6),

$$ \begin{aligned}[b] &\lim_{\varepsilon\rightarrow0} \int_{\varOmega}b(x,t)^{\frac {p(x,t)-1}{p(x,t)}} \bigl( \vert \nabla u \vert ^{p(x,t)-2}\nabla u- \vert \nabla v \vert ^{p(x,t)-2}\nabla v \bigr) \\ &\qquad\cdot b(x,t)^{\frac{1}{p(x,t)}}\bigl[\nabla(u_{\varepsilon }-v_{\varepsilon})- \nabla(u-v)\bigr] \bigl[h'_{n}(u_{\varepsilon}-v_{\varepsilon })-h'_{n}(u-v) \bigr] \,dx\,dt \\ &\quad=0. \end{aligned} $$
(4.8)

By (4.7)–(4.8), we have

$$ \begin{aligned}[b] &\lim_{\varepsilon\rightarrow0} \iint_{Q_{\tau s}}b(x,t)^{\frac {p(x,t)-1}{p(x,t)}} \bigl( \vert \nabla u \vert ^{p(x,t)-2}\nabla u- \vert \nabla v \vert ^{p(x,t)-2}\nabla v \bigr) \\ &\qquad\cdot b(x,t)^{\frac{1}{p(x,t)}}\bigl[\nabla(u_{\varepsilon }-v_{\varepsilon})- \nabla(u-v)\bigr]h'_{n}(u_{\varepsilon}-v_{\varepsilon }) \,dx\,dt \\ &\quad=\lim_{\varepsilon\rightarrow0} \iint_{Q_{\tau s}}b(x,t)^{\frac {p(x,t)-1}{p(x,t)}} \bigl( \vert \nabla u \vert ^{p(x,t)-2}\nabla u- \vert \nabla v \vert ^{p(x,t)-2}\nabla v \bigr) \\ &\qquad\cdot b(x,t)^{\frac{1}{p(x,t)}}\bigl[\nabla(u_{\varepsilon}-v_{\varepsilon })- \nabla(u-v)\bigr]h'_{n}(u-v) \,dx\,dt \\ &\qquad+\lim_{\varepsilon\rightarrow0} \iint_{Q_{\tau s}}b(x,t)^{\frac {p(x,t)-1}{p(x,t)}}(x) \bigl( \vert \nabla u \vert ^{p(x,t)-2}\nabla u- \vert \nabla v \vert ^{p(x,t)-2}\nabla v \bigr) \\ &\qquad\cdot b(x,t)^{\frac{1}{p(x,t)}}\bigl[\nabla(u_{\varepsilon }-v_{\varepsilon})- \nabla(u-v)\bigr] \bigl[h'_{n}(u_{\varepsilon}-v_{\varepsilon })-h'_{n}(u-v) \bigr] \,dx\,dt \\ &\quad=\lim_{\varepsilon\rightarrow0} \iint_{Q_{\tau s}}b(x,t)^{\frac {p(x,t)-1}{p(x,t)}} \bigl( \vert \nabla u \vert ^{p(x,t)-2}\nabla u- \vert \nabla v \vert ^{p(x,t)-2}\nabla v \bigr) \\ &\qquad\cdot b(x,t)^{\frac{1}{p(x,t)}}\bigl[\nabla(u_{\varepsilon}-v_{\varepsilon })- \nabla(u-v)\bigr]h'_{n}(u-v) \,dx\,dt \\ &\quad=0. \end{aligned} $$
(4.9)

In the third place,

$$ \begin{aligned}[b] &\lim_{\varepsilon\rightarrow0} \iint_{Q_{\tau s}}b(x,t) \bigl( \vert \nabla u \vert ^{p(x,t)-2}\nabla u- \vert \nabla v \vert ^{p(x,t)-2}\nabla v \bigr) \nabla(u_{\varepsilon}-v_{\varepsilon}) h'_{n}(u_{\varepsilon }-v_{\varepsilon})\,dx\,dt \\ &\quad= \iint_{Q_{\tau s}}b(x,t) \bigl( \vert \nabla u \vert ^{p(x,t)-2}\nabla u- \vert \nabla v \vert ^{p(x,t)-2}\nabla v \bigr) \nabla(u-v) h'_{n}(u-v)\,dx\,dt \\ &\quad\geq0. \end{aligned} $$
(4.10)

In the fourth place, since

$$\begin{gathered} \int_{\varOmega} \bigl\vert b(x,t)^{\frac{1}{p(x,t)}} (u_{\varepsilon} - v_{\varepsilon})_{x_{i}}h'_{n}(u_{\varepsilon} - v_{\varepsilon}) \bigr\vert ^{p(x,t)}\,dx \\ \quad\leq c(n) \int_{\varOmega} \bigl\vert b(x,t)^{\frac{1}{p(x,t)}} (u_{\varepsilon} - v_{\varepsilon})_{x_{i}} \bigr\vert ^{p(x,t)}\,dx\leq c(n),\end{gathered} $$

as \(\varepsilon\rightarrow0\), we have

$$b(x,t)^{\frac{1}{p(x,t)}}h'_{n}(u_{\varepsilon} - v_{\varepsilon}) (u_{\varepsilon}-v_{\varepsilon})_{x_{i}} \rightharpoonup b(x,t)^{\frac {1}{p(x,t)}}(u-v)_{x_{i}}h_{n}'(u-v), \quad\text{in } L^{1}\bigl(0,T; L^{p(x,t)}(\varOmega)\bigr). $$

By (1.12), \(|\sum_{i=1}^{N}g^{i}(x,t)|\leq cb(x,t)^{\frac {1}{p(x,t)}}\), we extrapolate that

$$\begin{gathered} \lim_{\varepsilon\rightarrow0} \iint_{Q_{\tau s}}\sum_{i=1}^{N} g^{i}(x,t)\bigl[\gamma_{i}(u) - \gamma_{i}(v) \bigr] (u_{\varepsilon} - v_{\varepsilon})_{x_{i}}h'_{n}(u_{\varepsilon} - v_{\varepsilon}) \,dx\,dt \\ \quad= \iint_{Q_{\tau s}} \sum_{i=1}^{N}g^{i}(x,t) \bigl[\gamma_{i}(u) - \gamma _{i}(v)\bigr] \cdot(u - v)_{x_{i}}h'_{n}(u-v)\,dx\,dt. \end{gathered} $$

Moreover, since

$$\bigl\vert \bigl[\gamma_{i}(u) -\gamma_{i}(v) \bigr]h_{n}'(u - v) \bigr\vert \leq c \bigl\vert (u-v)h_{n}'(u - v) \bigr\vert \leq c, $$

if we denote \(\varOmega_{1n}=\{x\in\varOmega: |u - v| < \frac{1}{n}\}\), we have

$$ \begin{aligned}[b] & \Biggl\vert \iint_{Q_{\tau s}} \sum_{i=1}^{N}g^{i}(x,t) \bigl[\gamma_{i}(u) -\gamma_{i}(v)\bigr]h'_{n}(u - v) (u - v)_{x_{i}}\,dx\,dt \Biggr\vert \\ &\quad= \Biggl\vert \int_{\tau}^{s} \int_{\varOmega_{1n}} \sum_{i=1}^{N}g^{i}(x,t) \bigl[\gamma_{i}(u) -\gamma_{i}(v)\bigr]h'_{n}(u - v) (u - v)_{x_{i}}\,dx\,dt \Biggr\vert \\ &\quad\leqslant c \int_{\tau}^{s} \int_{\varOmega_{1n}} \Biggl\vert \sum_{i=1}^{N}g^{i}(x,t) (u - v)_{x_{i}} \Biggr\vert \,dx\,dt \\ &\quad\leqslant c { \biggl[ \int_{\tau}^{s} \int_{\varOmega_{1n}} \bigl\vert b^{\frac{1}{p(x,t)}}\nabla(u - v) \bigr\vert ^{p(x,t)}\,dx\,dt \biggr]^{\frac{1}{p_{1}}}} \\ &\quad\leq c. \end{aligned} $$
(4.11)

If \(\varOmega_{1n}\) has 0 measure, from (4.11), letting \(n\rightarrow\infty\), we have

$$\lim_{n \to\infty} \int_{\varOmega_{1n}} \bigl\vert b^{\frac {1}{p(x,t)}}\nabla(u - v) \bigr\vert ^{p(x,t)}\,dx = 0. $$

While \(\varOmega_{1n}\) is with a positive measure, from (4.11), using the dominated convergence theorem, we directly have

$$\lim_{n \to\infty} \iint_{Q_{\tau s}} \sum_{i=1}^{N}g^{i}(x,t) \bigl[\gamma_{i}(u) -\gamma_{i}(v)\bigr]h_{n}(u - v) (u - v)_{x_{i}}\,dx\,dt = 0. $$

Therefore, we have

$$ \lim_{n \to\infty} \iint_{Q_{\tau s}} \sum_{i=1}^{N}g^{i}(x,t) \bigl[\gamma _{i}(u) - \gamma_{i}(v)\bigr] \cdot(u - v)_{x_{i}}h'_{n}(u-v)\,dx\,dt = 0. $$
(4.12)

Once more,

$$ \begin{aligned} &{-}\lim_{\varepsilon\rightarrow0}\sum _{i = 1}^{N} \iint_{Q_{\tau s}} \bigl[\gamma_{i}(u) - \gamma_{i}(v)\bigr]h_{n}(u_{\varepsilon} - v_{\varepsilon})\frac{\partial g^{i}(x,t)}{\partial x_{i}} \,dx\,dt \\ &\quad=-\sum_{i = 1}^{N} \iint_{Q_{\tau s}} \bigl[\gamma_{i}(u) - \gamma _{i}(v)\bigr]h_{n}(u - v)\frac{\partial g^{i}(x,t)}{\partial x_{i}} \,dx\,dt, \end{aligned} $$

and by assumption (i) \(\beta\leq1\), or (ii)

$$\bigl\vert \gamma_{i}(t_{1})-\gamma_{i}(t_{2}) \bigr\vert \leq c \bigl\vert \vert t_{1} \vert ^{\beta-1}t_{1}- \vert t_{2} \vert ^{\beta-1}t_{2} \bigr\vert , $$

we easily deduce that

$$ \begin{aligned}[b] &\lim_{n\rightarrow\infty} \Biggl\vert -\sum_{i = 1}^{N} \iint_{Q_{\tau s}} \bigl[\gamma_{i}(u) - \gamma_{i}(v)\bigr]h_{n}(u - v)\frac{\partial g^{i}(x,t)}{\partial x_{i}} \,dx\,dt \Biggr\vert \\ &\quad\leq c \iint_{Q_{\tau s}} \bigl\vert \vert u \vert ^{\beta-1}u- \vert v \vert ^{\beta-1}v \bigr\vert \,dx\,dt. \end{aligned} $$
(4.13)

Last but not least,

$$ \begin{aligned}[b] &\lim_{n\rightarrow\infty}\lim _{\varepsilon\rightarrow0} \iint _{Q_{\tau s}} h_{n}(u_{\varepsilon}- v_{\varepsilon})\frac{\partial ( \vert u \vert ^{\beta-1}u - \vert v \vert ^{\beta-1}v )}{\partial t}\,dx\,dt \\ &\quad=\lim_{n\rightarrow\infty} \iint_{Q_{\tau s}} h_{n}(u- v)\frac {\partial ( \vert u \vert ^{\beta-1}u - \vert v \vert ^{\beta-1}v )}{\partial t}\,dx\,dt \\ &\quad= \iint_{Q_{\tau s}}\operatorname{sign}(u-v)\frac{\partial ( \vert u \vert ^{\beta -1}u - \vert v \vert ^{\beta-1}v )}{\partial t}\,dx\,dt \\ &\quad= \iint_{Q_{\tau s}}\operatorname{sign} \bigl( \vert u \vert ^{\beta-1}u - \vert v \vert ^{\beta -1}v \bigr)\frac{\partial ( \vert u \vert ^{\beta-1}u - \vert v \vert ^{\beta -1}v )}{\partial t}\,dx\,dt \\ &\quad= \int_{\tau}^{s}\frac{d}{dt} \bigl\Vert \vert u \vert ^{\beta-1}u - \vert v \vert ^{\beta -1}v \bigr\Vert _{L^{1}(\varOmega)}\,dt. \end{aligned} $$
(4.14)

Then, by (4.5), (4.6), (4.7), (4.9), (4.10), (4.12), (4.14), we have

$$\int_{\tau}^{s}\frac{d}{dt} \bigl\Vert \vert u \vert ^{\beta-1}u - \vert v \vert ^{\beta -1}v \bigr\Vert _{L^{1}(\varOmega)}\,dt \leq c \iint_{Q_{\tau s}}\bigl| \vert u \vert ^{\beta-1}u- \vert v \vert ^{\beta-1}v\bigr|\,dx\,dt. $$

By the Gronwall inequality,

$$\begin{gathered} \int_{\varOmega}\bigl| \vert u \vert ^{\beta-1}u(x,s) - \vert v \vert ^{\beta-1}v(x,s) \bigr|\,dx \\\quad\leqslant \int_{\varOmega}\bigl| \bigl\vert u(x,\tau) \bigr\vert ^{\beta -1}u(x,\tau) - \bigl\vert v(x,\tau) \bigr\vert ^{\beta-1}v(x,\tau) \bigr|\,dx,\quad \forall t \in[0,T).\end{gathered} $$

By the arbitrariness of τ, we extrapolate that

$$\int_{\varOmega}\bigl| \vert u \vert ^{\beta-1}u(x,s) - \vert v \vert ^{\beta-1}v(x,s) \bigr|\,dx \leqslant \int_{\varOmega}\bigl| \vert u_{0} \vert ^{\beta-1}u_{0} - \vert v_{0} \vert ^{\beta-1}v_{0} \bigr|\,dx,\quad\forall s \in[0,T). $$

The proof is complete. □

References

  1. Antontsev, S., Rodrigues, J.F.: On stationary thermo-rheological viscous flows. Ann. Univ. Ferrara, Sez. 7: Sci. Mat. 52, 19–36 (2006)

    Article  MathSciNet  Google Scholar 

  2. Rajagopal, K., Ruzicka, M.: Mathematical modelling of electro-rheological fluids. Contin. Mech. Thermodyn. 13, 59–78 (2001)

    Article  Google Scholar 

  3. Acerbi, E., Mingione, G.: Regularity results for stationary electro-rheological fluids. Arch. Ration. Mech. Anal. 164, 213–259 (2002)

    Article  MathSciNet  Google Scholar 

  4. Chen, Y., Levine, S., Rao, M.: Variable exponent, linear growth functionals in image restoration. SIAM J. Appl. Math. 66, 1383–1406 (2006)

    Article  MathSciNet  Google Scholar 

  5. Aboulaich, R., Meskine, D., Souissi, A.: New diffusion models in image processing. Comput. Math. Appl. 56, 874–882 (2008)

    Article  MathSciNet  Google Scholar 

  6. Levine, S., Chen, Y.M., Stanich, J.: Image restoration via nonstandard diffusion. Department of Mathematics and Computer Science, Duquesne University (2004)

  7. Guo, B., Li, Y.J., Gao, W.J.: Singular phenomena of solutions for nonlinear diffusion equations involving \(p(x)\)-Laplace operator and nonlinear source. Z. Angew. Math. Phys. 66, 989–1005 (2015)

    Article  MathSciNet  Google Scholar 

  8. Antontsev, S., Shmarev, S.: Parabolic equations with anisotropic nonstandard growth conditions. Int. Ser. Numer. Math. 154, 33–44 (2007)

    Article  MathSciNet  Google Scholar 

  9. Antontsev, S., Shmarev, S.: Anisotropic parabolic equations with variable nonlinearity. Publ. Math. 53, 355–399 (2009)

    Article  MathSciNet  Google Scholar 

  10. Antontsev, S., Shmarev, S.: Extinction of solutions of parabolic equations with variable anisotropic nonlinearities. Proc. Steklov Inst. Math. 261, 11–22 (2008)

    Article  MathSciNet  Google Scholar 

  11. Antontsev, S., Shmarev, S.: Vanishing solutions of anisotropic parabolic equations with variable nonlinearity. J. Math. Anal. Appl. 361, 371–391 (2010)

    Article  MathSciNet  Google Scholar 

  12. Antontsev, S., Chipot, M., Shmarev, S.: Uniqueness and comparison theorems for solutions of doubly nonlinear parabolic equations with nonstandard growth conditions. Commun. Pure Appl. Anal. 12, 1527–1546 (2013)

    Article  MathSciNet  Google Scholar 

  13. Antontsev, S., Shmarev, S.: Doubly degenerate parabolic equations with variable nonlinearity II: blow-up and extinction in a finite time. Nonlinear Anal. 95, 483–498 (2014)

    Article  MathSciNet  Google Scholar 

  14. Gao, Y.C., Chu, Y., Gao, W.J.: Existence, uniqueness, and nonexistence of solution to nonlinear diffusion equations with \(p(x, t)\)-Laplacian operator. Bound. Value Probl. 2016, Article ID 149 (2016)

    Article  MathSciNet  Google Scholar 

  15. Liu, B., Dong, M.: A nonlinear diffusion problem with convection and anisotropic nonstandard growth conditions. Nonlinear Anal., Real World Appl. 48, 383–409 (2019)

    Article  MathSciNet  Google Scholar 

  16. Ye, H., Yin, J.: Propagation profile for a non-Newtonian polytropic filtration equation with orientated convection. J. Math. Anal. Appl. 421, 1225–1237 (2015)

    Article  MathSciNet  Google Scholar 

  17. Al-Bayati, S.A., Worbel, L.C.: Radial integration boundary element method for two-dimensional non-homogeneous convection–diffusion–reaction problems with variable source term. Eng. Anal. Bound. Elem. 101, 89–101 (2019)

    Article  MathSciNet  Google Scholar 

  18. Marcellini, P.: A variational approach to parabolic equations under general and \(p,q\)-growth conditions. Nonlinear Anal. 194, Article ID 111456 (2020)

    Article  MathSciNet  Google Scholar 

  19. Zeng, S., Gasiński, L., Winkert, P., Bai, Y.: Existence of solutions for double phase obstacle problems with multivalued convection term. J. Math. Anal. Appl. (2020). https://doi.org/10.1016/j.jmaa.2020.123997

    Article  Google Scholar 

  20. Zhan, H., Feng, F.: Solutions of evolutionary \(p(x)\)-Laplacian equation based on the weighted variable exponent space. Z. Angew. Math. Phys. 68, Article ID 134 (2017)

    Article  MathSciNet  Google Scholar 

  21. Zhan, H., Feng, Z.: Solutions of evolutionary equation based on the anisotropic variable exponent Sobolev space. Z. Angew. Math. Phys. 70, Article ID 110 (2019)

    Article  MathSciNet  Google Scholar 

  22. Zhan, H.: The weak solutions of an evolutionary \(p(x)\)-Laplacian equation are controlled by the initial value. Comput. Math. Appl. 76, 2272–2285 (2018)

    Article  MathSciNet  Google Scholar 

  23. Zhan, H.: A new kind of the solutions of a convection–diffusion equation related to the \(p(x)\)-Laplacian. Bound. Value Probl. 2017, Article ID 117 (2017)

    Article  MathSciNet  Google Scholar 

  24. Zhan, H., Wen, J.: Evolutionary \(p(x)\)-Laplacian equation free from the limitation of the boundary value. Electron. J. Differ. Equ. 2016, Article ID 143 (2016)

    Article  MathSciNet  Google Scholar 

  25. Zhan, H., Feng, Z.: Partial boundary value condition for a nonlinear degenerate parabolic equation. J. Differ. Equ. 267, 2874–2890 (2019)

    Article  MathSciNet  Google Scholar 

  26. Xu, R.Z., Su, J.: Global existence and finite time blow-up for a class of semilinear pseudo-parabolic equations. J. Funct. Anal. 264, 2732–2763 (2013)

    Article  MathSciNet  Google Scholar 

  27. Chen, H., Tian, S.: Initial boundary value problem for a class of semilinear pseudo-parabolic equations with logarithmic nonlinearity. J. Differ. Equ. 258(12), 4424–4442 (2015)

    Article  MathSciNet  Google Scholar 

  28. Cao, Y., Wang, Z., Yin, J.: A semilinear pseudo-parabolic equation with initial data non-rarefied at ∞. J. Funct. Anal. 277(10), 3737–3756 (2019)

    Article  MathSciNet  Google Scholar 

  29. Rădulescu, V.D., Repovš, D.D.: Partial Differential Equations with Variable Exponents: Variational Methods and Qualitative Analysis. CRC Press, Boca Raton (2015)

    Book  Google Scholar 

  30. Giacomoni, J., Rădulescu, V.D., Warnault, G.: Quasilinear parabolic problem with variable exponent: qualitative analysis and stabilization. Commun. Contemp. Math. 20(8), Article ID 1750065 (2018)

    Article  MathSciNet  Google Scholar 

  31. Afrouzi, G.A., Mirzapour, M., Rădulescu, V.D.: Qualitative analysis of solutions for a class of anisotropic elliptic equations with variable exponent. Proc. Edinb. Math. Soc. 59, 541–557 (2016)

    Article  MathSciNet  Google Scholar 

  32. Saiedinezhad, S., Rădulescu, V.D.: Multiplicity results for a nonlinear Robin problem with variable exponent. J. Nonlinear Convex Anal. 17(8), 1567–1582 (2016)

    MathSciNet  MATH  Google Scholar 

  33. Mihăilescu, M., Rădulescu, V.D., Tersian, S.: Homoclinic solutions of difference equations with variable exponents. Topol. Methods Nonlinear Anal. 38(2), 277–289 (2011)

    MathSciNet  MATH  Google Scholar 

  34. Chen, C., Wang, R.: Global existence and \(L^{\infty}\) estimates of solution for doubly degenerate parabolic equation. Acta Math. Sin. 44, 1089–1098 (2001) (in Chinese)

    MathSciNet  MATH  Google Scholar 

  35. Wu, Z., Zhao, J., Yin, J., Li, H.: Nonlinear Diffusion Equations. Word Scientific, Singapore (2001)

    Book  Google Scholar 

  36. Fan, X.L., Zhao, D.: On the spaces \({L^{p(x)}(\varOmega)}\) and \({W^{m,p(x)}}\). J. Math. Anal. Appl. 263, 424–446 (2001)

    Article  MathSciNet  Google Scholar 

  37. Kovácik, O., Rákosník, J.: On spaces \({L^{p(x)}}\) and \({W^{k,p(x)}}\). Czechoslov. Math. J. 41, 592–618 (1991)

    MATH  Google Scholar 

Download references

Acknowledgements

The author would like to thank everyone for their kind help.

Availability of data and materials

No applicable.

Funding

The paper is supported by the Natural Science Foundation of Fujian province (2019J01858), supported by the Science Foundation of Xiamen University of Technology, China.

Author information

Authors and Affiliations

Authors

Contributions

The author read and approved the final manuscript.

Corresponding author

Correspondence to Huashui Zhan.

Ethics declarations

Competing interests

The author declares that he has no competing interests.

Rights and permissions

Open Access This article is licensed under a Creative Commons Attribution 4.0 International License, which permits use, sharing, adaptation, distribution and reproduction in any medium or format, as long as you give appropriate credit to the original author(s) and the source, provide a link to the Creative Commons licence, and indicate if changes were made. The images or other third party material in this article are included in the article’s Creative Commons licence, unless indicated otherwise in a credit line to the material. If material is not included in the article’s Creative Commons licence and your intended use is not permitted by statutory regulation or exceeds the permitted use, you will need to obtain permission directly from the copyright holder. To view a copy of this licence, visit http://creativecommons.org/licenses/by/4.0/.

Reprints and permissions

About this article

Check for updates. Verify currency and authenticity via CrossMark

Cite this article

Zhan, H. The nonnegative weak solution of a degenerate parabolic equation with variable exponent growth order. Bound Value Probl 2020, 69 (2020). https://doi.org/10.1186/s13661-020-01364-x

Download citation

  • Received:

  • Accepted:

  • Published:

  • DOI: https://doi.org/10.1186/s13661-020-01364-x

MSC

Keywords